+1 Daumen
426 Aufrufe

Aufgabe:

Die Funktion g(x) = ln(1+x) lässt sich in die Mac Laurin-Reihe\( \sum\limits_{n=0}^{\infty}{\frac{(-1)^n}{n+1}}x^{n+1} \) mit -1<x<1 entwickeln. Bis zur wievielten Ordnung muss die Reihe entwickelt werden, wenn der Fehler auf jeden Fall kleiner als 10(-4) sein soll?

Avatar von

Meinst Duz im gesamten Bereich von -1< x < 1

Ich glaube schon. Die Aufgabe wurde so bei einer Prüfung gestellt und so habe ich sie interpretiert.

2 Antworten

0 Daumen
 
Beste Antwort

Schau al hier https://mathepedia.de/Logarithmusfunktion.html

Im ersten Fall kann man eine Abschätzung des Restgliedes vornehmen und kommt auf $$  |R_n(x)| \le \frac{1}{n+1} $$ und damit ist die Approximation genauer als \( 10^{-4} \) wenn \( n > 10^4 -1 \) gilt. Das ist aber nicht die kleinste obere Schranke.

Im zweiten Fall ist das Restglied kleiner als $$  \left| \frac{x^{n+1}}{1+x} \right|$$ daraus sieht man, dass es in der Nähe von \( -1 \) der von abakus beschriebene Fall eintritt.

Avatar von 39 k
0 Daumen

Der Fehler ist natürlich am Rand größer, und die Konvergenz geht für positive x schneller als für negative x (Leibniz geht/geht nicht). Am langsamsten ist die Konvergenz in der Nähe von x=-1, und bei Annäherung an -1 muss n unendlich groß werden.

Die Frage lässt sich nicht mit einem festen Wert für n beantworten.

Avatar von 53 k 🚀

Ein anderes Problem?

Stell deine Frage

Willkommen bei der Mathelounge! Stell deine Frage einfach und kostenlos

x
Made by a lovely community